Find the area of a triangle with legs that are: 12 m, 15 m, and 9 m.

Answers

Answer 1

Answer:

108 meters squared or m^2

Step-by-step explanation:

* means multiply

15 is probably hypotenuse because its the longest

12 and 9 are probably base and height

area = base * height

area = 12 * 9

area = 108

Answer 2

Answer:

54m^2

Step-by-step explanation:


Related Questions

According to a study, the probability that a randomly selected teenagar shopped at a mall at least once during a week was 0.61. Let X be the number of students in a randomly selected group of 50 that will shop at a mall during the next week. (a) Compute the expected value and standard deviation of X. expected value standard deviation (b) Fill in the missing quantity. (Round your answer to the nearest whole number.)There is an approximately 2.5% chance that _____ or more teenagers in the group will shop at the mall during the next week.

Answers

Answer:

Step-by-step explanation:

Given that:

p = 0.61

If X is the the number of students in a randomly selected group  of a sample size n = 50

The expected value and the standard deviation can be computed as follows:

The expected value E(X) = np

The expected value E(X) =  50 × 0.61

The expected value E(X) = 30.5

The required standard deviation = [tex]\sqrt{np(1-p)}[/tex]

The required standard deviation = [tex]\sqrt{30.5(1-0.61)}[/tex]

The required standard deviation = [tex]\sqrt{30.5(0.39)}[/tex]

The required standard deviation = [tex]\sqrt{11.895}[/tex]

The required standard deviation = 3.4489

The required standard deviation =  3.45

(b) Fill in the missing quantity. (Round your answer to the nearest whole number.)

There is an approximately 2.5% chance that _____ or more teenagers in the group will shop at the mall during the next week.

From the given information:

Now, we can deduce that:

the mean = 30.5

standard deviation = 3.45

Using the empirical rule:

At 95% confidence interval;

[μ - 2σ,  μ + 2σ] = [ 30.5 - 2(3.45) , 30.5 + 2(3.45)]

[μ - 2σ,  μ + 2σ] =  [ 30.5 - 6.9 , 30.5 + 6.9]

[μ - 2σ,  μ + 2σ] = [ 23.6, 37.4]

The 2.5% of the observations are less than 95% confidence interval and 2.5% observations are greater than 95% confidence interval.

The required number of teenagers is = the upper limit of the 95% confidence interval = 37

There is an approximately 2.5% chance that __37___ or more teenagers in the group will shop at the mall during the next week.

simplify theta(sin theta) ²/2tan theta​

Answers

Step-by-step explanation:

When θ is very small, θ ≈ sin θ ≈ tan θ.

θ (sin θ)² / (2 tan θ)

θ³ / (2θ)

θ² / 2

What is the present value of $25,000 to be received in 15 years at an (a) 6.2% interest rate and, (b) 9.6% interest rate? Explain why the present value is lower when the interest rate is higher.

Answers

Answer:

With a higher interest rate, you get more money per period, that acuminates.

The Present value is the starting amount... If the final amount is the same in both scenarios, and you get more interest in the second situation, you can start with a lower amount

Step-by-step explanation:

[tex]PV = CF / (1 + r) t[/tex]

pv1 = 25000/(1.062)^15 = $10,140.80

pv2 = 25000/(1.096)^15 = $6,320.94

Instructions: The polygons in each pair are similar. Find the
missing side length.

Answers

Answer:

missing side length is 12

Step-by-step explanation:

similar means that all correlating pairs of sides have the same ratio (old side length) / (new side length).

so, when we know the ratio of one pair, we can apply it to any other side to calculate the correlating side.

we see, when we go from small to large, that we have the ratio 32/40 = 4/5.

so, multiplying the larger side by this, we get the shorter side.

15 × 4/5 = 3 × 4 = 12

the proportion part in your picture is a bit confusing :

yes,

x/15 = 32/40 = 32/40

I don't know, why this last expression was repeated.

x/15 = 32/40

x = 15×32/40 = 15×4/5 = 3×4 = 12

as you see we get of course the same result doing it that way.

Find the quotient of 90 over -10

Answers

90/-10

= 9/-1

= -9

So, -9 is the quotient.

Hi! The answer is -9 because 90/-10 = -9
I hope this helps you, Goodluck! :)

The graph shown is the solution set for which of the following inequalities?

Answers

Answer:

b is your answer hope it is helpful

Answer:

b is the correct answer

Step-by-step explanation:

the answer is b

PLEASE HELP WITH BOTH OF THESE ASAP!

Answers

Answer:

Hello,

Step-by-step explanation:

[tex]\overline{x}=3262\\\sigma=1100\\\\P(x\geq 1000)=P(x\leq 3262+(3262-1000))=P(x\leq 5524)\\\\z=\dfrac{x-\overline{x}}{\sigma} =\dfrac{5524-3262}{1100} =2.05\overline{63}\\\\\\P(z\leq 2.05)=0.9789\\\\P(z\leq 2.06)=0.9803\\\\\\\boxed{0.9789\leq P\leq 0.9803}\\\\\\[/tex]

[tex]P(x\leq 4000)=P(z\leq \dfrac{4000-3262}{1100} )=P(z\leq 0.67\overline{09})\\\\P(z\leq 0.67)=0.7486\\P(z\leq 0.68)=0.7517\\\\0.7486\leq P\leq 0.7517\\[/tex]

p-value problem. Suppose the director of manufacturing at a clothing factory needs to determine wheteher a new machine is producing a particulcar type of cloth according to the manufacturer s specification which indicate that the cloth should have mean breaking strength of 70 pounds and a standard deviation of 3.5 pounds. A sample of 49 pieces reveals a sample mean of 69.1 pounds. THe p value for this hypothesis testing scenario is

Answers

Answer:

The P-Value is  0.07186  

Step-by-step explanation:

GIven that :

Mean = 70

standard deviation = 3.5

sample size n = 49

sample mean = 69.1

The null hypothesis and the alternative hypothesis can be computed as follows;

[tex]H_o : \mu = 70 \\ \\ H_1 : \mu \neq 70[/tex]

The standard z score formula can be expressed as follows;

[tex]\mathtt{z = \dfrac{\overline X - \mu}{\dfrac{\sigma}{\sqrt{n}}}}[/tex]

[tex]\mathtt{z = \dfrac{69.1 - 70}{\dfrac{3.5}{\sqrt{49}}}}[/tex]

[tex]\mathtt{z = \dfrac{-0.9}{\dfrac{3.5}{7}}}[/tex]

z = -1.8

Since the test is two tailed and using the Level of significance = 0.05

P- value = 2 × P( Z< - 1.8)

From normal tables

P- value = 2 × (0.03593)

The P-Value is  0.07186  

Using a rating scale, Tekinarslan (2008) measured computer anxiety among university students who use the computer very often, often, sometimes, and seldom. Below are the results of the one-way ANOVA. Source of Variation SS df MS F Between groups 1,959.79 3 653.26 21.16* Within groups (error) 3,148.61 102 30.86 Total 5,108.41 105 (a) What are the values for N and k

Answers

Answer:

k = 4 ; N = 106

Step-by-step explanation:

Given the result of the one way ANOVA :

- - - - - - - - - - - - - - - SS - - - - df - - MS - - - - - F

Between groups - 1,959.79 - 3 - - 653.26 - 21.16*

Error - - - - - - - - - - 3,148.61 - -102 --30.86

Total - - - - - - - - - - 5,108.41 - 105

To obtain the value of 'k' which is the number of groups observed :

The degree of freedom between groups or degree of freedom of treatment (DFT) is obtained by the formula:

Number of observed groups(k) - 1

DFT = k - 1

From the ANOVA result ; degree of freedom between groups = 3

Hence,

3 = k - 1

k = 3 +1 = 4

Hence, number of observed groups = 4

To obtain N;

N is related to k and the degree of freedom Error (DFE)

DFE = N - k

From the ANOVA result, DFE = 102 and k = 4

102 = N - 4

102 + 4 = N

N = 106

You and your friend decide to get your cars inspected. You are informed that ​83% of cars pass inspection. If the event of your​ car's passing is independent of your​ friend's car. please help me with d! an explanation with steps would be nice but mandatory

Answers

It has to be d because this app sucks

Find the length of side
x in simplest radical form with a rational denominator.



Thanks in advance

Answers

Answer:

2*√2

Step-by-step explanation:

You need to use the trigonometrical function of angle (You should use the side sqrt2 and x to find the x). Take any acute angle (for example, 60 degrees)

The cos of this angle is equal to the ratio of adjoined leg (sqrt2) to the hypotenude (the side opposite to right angle, it is x)

cos60degrees= sqrt2/ x

1/2= sqrt2/x

x= 2*sqrt2= 2*√2

Which equation represents this statement: A number minus 6 is 168? 6 − n = 168 n ÷ 6 = 168 n − 6 = 168 6n = 168

Answers

Answer:n-6=168

Step-by-step explanation:

The statement starts with the variable first.

Answer:

n - 6 = 168.

Step-by-step explanation:

Let's say that the value of the number is n.

n minus 6 is 168, so n - 6 = 168.

n - 6 = 168

n = 174.

Hope this helps!

Please Help me with this Click to select the following graphic figure. A square circumscribed about a circle:

Answers

The answer would be the first image.

Step-by-step explanation:

From context, it appears that to be circumscribed is to be drawn about; thus the square circumscribed about the circle is the first graph.

Answer:

The first image which is a circle in a square

Choose the correct simplification of
fh23
fh17

Answers

Answer:

C

Step-by-step explanation:

Answer:

f^6h^6

Step-by-step explanation:

f^9 h^23

-----------------

f^3 h^17

We know a^b / a^c = a^(b-c)

f^9 / f^3 = f^(9-3) = f^6

h^23 / h^17 = h^(23-17) = h^6

f^6h^6

An octagonal pyramid ... how many faces does it have, how many vertices and how many edges? A triangular prism ... how many faces does it have, how many vertices and how many edges? a triangular pyramid ... how many faces does it have, how many vertices and how many edges?

Answers

1: 8 faces and 9 with the base 9 vertices and 16 edges

2: 3 faces and 5 with the bases 6 vertices and 9 edges

3: 3 faces and 4 with the base 4 vertices and 6 edges

Hope this can help you.

1: 8 faces and 9 with the base 9 vertices and 16 edges

2: 3 faces and 5 with the bases 6 vertices and 9 edges

3: 3 faces and 4 with the base 4 vertices and 6 edges

I got a negative number as an answer for this sine rule question and I don't know what did I do wrong​

Answers

Answer:

x=14.42192

Step-by-step explanation:

Angle A = 180 - 86 =94

The sum of the angles of a triangle is 180

sin A        sin 46

------- = -----------

20            x

sin 94 * x = 20 sin 46

x = 20 sin 46 / sin 94

x=14.42192

Please help . I’ll mark you as brainliest if correct!

Answers

Answer:

Stocks = $15,500

Bonds = $107,250

CD's = $47,250

Step-by-step explanation:

S + B + C = 170000

.0325S + .038B .067C = 7745

60,000 + C = b

S = $15,500

B = $107,250

C = $47,250

Which of the following statements is TRUE about the stepwise selection procedure?
A. The stepwise selection procedure uses Adjusted R-square as the "best" model criterion.
B. Backward stepwise procedure and forward stepwise procedure would end up with the same "best" model.
C. The "best" model determined by the stepwise selection method is the same model as what would be selected by complete search but stepwise method is usually faster.
D. Different choices of alpha limits for variable selection may end up with different final models.

Answers

Answer:

A. The stepwise selection procedure uses Adjusted R-square as the "best" model criterion.

Step-by-step explanation:

Stepwise regression is a model which uses variables in step by step manner. The procedure involves removal or inclusion of independent variables one by one. It adds the most significant independent variable and removes the less significant independent variable. Usually stepwise selection uses R-square or Mallows Cp for picking the best fit.

Construct a polynomial function with the following properties: third degree, only real coefficients, −3 and 3+i are two of the zeros, y-intercept is −90.

Answers

Answer:

[tex]\boxed{-3(x+3)(x^2-6x+10)}[/tex]

Step-by-step explanation:

Hello,

As the polynomial has only real coefficients, it means that 3-i is a zero too, because we apply the Conjugate Zeros Theorem.

It means that we can write the expression as below, k being a real number that we will have to identify.

[tex]k(x+3)(x-3-i)(x-3+i)=k(x+3)((x-3)^2-i^2)\\\\=k(x+3)(x^2-6x+9+1)\\\\=k(x+3)(x^2-6x+10)[/tex]

And for x = 0, y = -90 so we can write

-90=k*3*10, meaning that k=-3

Hope this helps.

Do not hesitate if you need further explanation.

Thank you

The size of a television is the length of the diagonal of its screen in inches. The aspect ratio of the screens of older televisions is 4:3, while the aspect ratio of newer wide-screen televisions is 16:9. Find the width and height of an newer 75-inch television whose screen has an aspect ratio of 16:9

Answers

Answer:

The Width = 65.44 inches

The Height = 36.81 inches

Step-by-step explanation:

We are told in the question that:

The width and height of an newer 75-inch television whose screen has an aspect ratio of 16:9

Using Pythagoras Theorem we known that:

Width² + Height² = Diagonal²

Since we known that the size of a television is the length of the diagonal of its screen in inches.

Hence, for this new TV

Width² + Height² = 75²

We are given ratio: 16:9 as aspect ratio

Width = 16x

Height = 9x

(16x)² +(9x)² = 75²

= 256x² + 81x² = 75²

337x² = 5625

x² = 5625/337

x² = 16.691394659

x = √16.691394659

x = 4.0855103303

Approximately x = 4.09

For the newer 75 inch tv set

The Height = 9x

= 9 × 4.09

= 36.81 inches

The Width = 16x

= 16 × 4.09

= 65.44 inches.

Find the sum of ∑4/k=1 (-4k)

Answers

Answer:

Hello,

Answer C: -40

Step-by-step explanation:

[tex]\displaystyle \sum_{k=1}^4\ (-4k)\\\\=-4*\sum_{k=1}^4\ k\\\\=-4*4*\dfrac{1+4}{2} \\\\=-4*5*2\\\\=-40\\[/tex]

2. Use the diagram and given information to answer the questions and prove the statement.

a. Re-draw the diagram of the overlapping triangles so that the two triangles are separated.

b. What additional information would be necessary to prove that the two triangles, XBY and ZAY , are congruent? What congruency would be applied?

c. Prove (AZ) is congruent to (BX) using a flow chart proof. ( ):both have a line over them

Answers

[tex] \huge{ \underline{ \tt{ \purple{Solution:}}}}[/tex]

2) a)⚘ Refer to the attachment....

After separating, we will get two triangles △XYB and △ZYA where ∠Y is common to both the triangles, hence their measure is equal. This will be use in further proof.

b) We have,

∠X = ∠Z (Given, ATQ)∠Y = common to both triangles. XY = ZY

So, here

Two pairs of corresponding angles are equal along the side contained between them. So, The above triangles are congurent by ASA criterion.

✤ No more additional information Required to prove the above triangles be congurent.

△XYB ≅ △ZYA (By ASA Criterion)

c) By using flow chart proof:

[tex] \boxed{ \sf{ \angle X = \angle Z}} \searrow[/tex]

[tex] \boxed{ \sf{\small{ \angle Y = com.}}} \rightarrow \boxed{\small{ \sf{ \triangle XYB \cong \triangle ZYA}}}\rightarrow \small{\boxed{ \sf{AZ= XB}}}[/tex]

[tex] \boxed{ \sf{XY = ZY}} \nearrow[/tex]

━━━━━━━━━━━━━━━━━━━━

Step-by-step explanation:

Hey mate ut answer is in the given attachment.

hope i help u

Evaluate the expression when a= 2 and c= -7. C-4a answer​

Answers

Answer:

-15

Step-by-step explanation:

Let a = 2 and c = -7.

[tex]c-4a\\(-7)-4(2)\\-7-8\\-15[/tex]

Solution:-

we know that the value of a is 2 and c is -7

Expression-

c-4a

= (-7)-4×2

= (-7)-8

= -15

~

Jerry walked a dog from 6:40 a.m. to 7:30 a.m. one day. If he was paid at the rate of $6 per hour, how much did he cam that day?

Answers

Answer: 5$

Explanation:

1hr = 6$ but 1hr = 60min
Or 60min = 6$

If he walk the dog for 50min:

60/50 = 6/x
X = (6 x 50)/60
X = 300/60
X = 5$

X is the amount he was paid for walking the dog for 50 min.

And 7:30 - 6:40 = 50min

Armando is baking 36 batches of brownies for the bake sale. Each batch of brownies takes cups of flour. What is a reasonable estimate of the amount of flour that he will need to bake all thirty-six batches of brownies?

Answers

Answer:

Well, let's assume that "cups" = 3 cups of flour.

Step-by-step explanation:

First, multiply 3x36.

If for some reason this is incorrect, try 2 cups instead of 3. Both are reasonable measurements when it comes to baking.

PLEASE SHOW YOUR WORK! I NEED THIS QUICK! THANK YOU!! POINTS AND RATES WILL BE GIVEN!!

Answers

Answer:

3

Step-by-step explanation:

Y is given by sin(theta). sin(-60)=-sqrt(3)/2

WHAT IS X³-27 SIMPLIFIED

Answers

Answer:

It is (x - 3)³ - 9x(3 - x)

Step-by-step explanation:

Express 27 in terms of cubes, 27 = 3³:

[tex] = {x}^{3} - {3}^{3} [/tex]

From trinomial expansion:

[tex] {(x - y)}^{3} = (x - y)(x - y)(x - y) \\ [/tex]

open first two brackets to get a quadratic equation:

[tex] {(x - y)}^{3} = ( {x}^{2} - 2xy + {y}^{2} )(x - y)[/tex]

expand further:

[tex] {(x - y)}^{3} = {x}^{3} - y {x}^{2} - 2y {x}^{2} + 2x {y}^{2} + x {y}^{2} - {y}^{3} \\ {(x - y)}^{3} = {x}^{3} - {y}^{3} + 3x {y}^{2} - 3y {x}^{2} \\ {(x - y)}^{3} = {x}^{3} - {y}^{3} + 3xy(y - x) \\ \\ { \boxed{( {x}^{3} - {y}^{3} ) = {(x - y)}^{3} - 3xy(y - x)}}[/tex]

take y to be 3, then substitute:

[tex]( {x}^{3} - 3^3) = {(x - 3)}^{3} - 9x(3 - x)[/tex]

the difference between a number and 6

Answers

Step-by-step explanation:

=The difference between a number and 6

the term difference is used in mathmatics as subtraction

let x = a number

x - 6 = the difference between that unknown number and 6

Answer:

Step-by-step explanation:

difference means subtract. Difference is always worded the same way. What you have written means that you start with an algebraic letter and then comes the minus sign and then the number

Let the number = x

x - 6

If it was worded as difference between 6 and a number, you would write it this way

6 - x

Complete the equation: x^ 2 +6x+__ =(_)^ 2

Answers

Answer:

x^2+6x+9= (x+3)^2

Step-by-step explanation:

x^ 2 +6x+__ =(_)^ 2

The formula is a^2+2ab+b^2= (a+b)^2

x^2= a^2

x=a

6x=2ab where a=x

6x=2x*b

b=3

b^2=9

x^2+6x+9= (x+3)^2

in a village in hawaii, about 80% of the residents are of hawaiian ancestry. Let n be the number of people you meet until you encounter the 1st person of hawaiian ancestry in the village. write a formula for the probability distribution

Answers

Answer:

The formula for the probability distribution is:

P(X = n) = q^(n - 1)p

= [0.2^(n - 1)]0.8

Step-by-step explanation:

This is a geometric probability distribution.

The probability of success p = 80% = 0.8

The probability of failure is q = 1 - p = 0.2

The formula is:

P(X = n) = q^(n - 1)p

= [0.2^(n - 1)]0.8

Other Questions
Based on your knowledge of word parts, which sentence uses the word introspection correctly?O A. Because of his experience as a firefighter, Matthew provided a unique introspection.OB. Gillian stood up straight and braced herself for her mother's introspection of her outfit..Samuel waited patiently for the contractors to come and perform the building introspection.ODAfter some soul-searching and introspection, Peter knew that it was time for him to move on. Examine four factors that have affected tha traditional roles of Ghanian woman what's stock exchange? PLEASE HELP PLEASE WILL MARK BRAINLY Under the NASAA model custody rule, an investment adviser would be considered to have custody of client assets if that adviser inadvertently receives:i. a check from a client from a client made out to the IA and does not return the check within 24 hoursii. a check from a client made out to a third party and Does not forward the check within 3 business daysiii. stock certificates from a client and does not forward them within 3 business daysiv. stock certificates from a client and does noT return them within 3 business days the ways in which the Greeks and Romans have influenced todays world. (Algebra) PLZ HELP ASAP! 4. Gives suggestions in two ways for the following problems.i. Your pronunciation is not so good.ii. You are putting on weightiii. You have forgotten your homeworkiv. You have no pen to write withv. You lost your note copy in the school Karl from Accounting is in a panic. He is convinced that he has identified malware on the serversa type of man-in-the-middle attack in which a Trojan horse manipulates calls between the browser and yet still displays back the user's intended transaction. What type of attack could he have stumbled on? What is the length of ADA.17B.15C.7D.1 Charlie's flower bed has a length of 4 feet and a width of four sixths feet. Which of the following is true1 The area of the flower bed is equal to 6 square feet.2The area of the flower bed is larger to 6 square feet.3 The area of the flower bed is equal to 4 square feet4 The area of the flower bed is smaller than 4 square feet. Find the equation of a parabola that has a vertex (3,5) and passes through the point (1,13).Oy= -27 - 3)' +5Oy=2(x + 3) - 5Oy=2(0 - 3)' + 5Oy= -3(2 3) + 5PLEASE HELP ME!! In the nation of Gondor, the EPA requires that half the new cars sold will meet a certain particulate emission standard a year later. A sample of 64 one-year-old cars revealed that only 24 met the particulate emission standard. The test statistic to see whether the proportion is below the requirement is Why must you include the water soluble vitamins in your daily diet? They cannot be stored by the body because they are water soluble and are eliminated daily. They help carry oxygen in the blood They help digest fats. They help build body tissue. Menciona 3 ideas para evitar la contaminacion y preservar el medio ambiente Find the equation of a line given the point and slope below. (6, 6) no slope A man starts repaying a loans with first insfallameny of rs.10 .If he increases the instalment by Rs 5 everything months, what amount will be paid by him in the 30the instalment. Lee lo siguiente. La Mezquita de Crdoba _[blank 1]_ patrimonio mundial desde 1984. Se _[blank 2]_ mucho en el parque de diversiones? El restaurante que te _[blank 3]_ es de dos tenedores. _[blank 4]_ este plato tpico de Mxico? Empareja los espacios en blanco con los verbos correctos.options:Habis recorridoha estadohe recomendadohan divertidoHas probadoha sido You want to save $98,000 to buy an boat by making an equal, end of year payment into a brokerage account for the next 9 years. If you expect to earn an annual interest rate of 7.75% on your account, how much do you need to deposit each year into your account? Why is environmental science important?